Difference between revisions of "1989 OIM Problems/Problem 6"
(Created page with "== Problem == Prove that there's and infinity of pairs of natural numbers that satisfy the equation: <math>2x^2-3x=3y^2</math> ~translated into English by Tomas Diaz. ~orders...") |
(solution) |
||
Line 5: | Line 5: | ||
== Solution == | == Solution == | ||
− | {{solution}} | + | We claim that there do <b>not</b> exist any solutions to this equation over the positive integers. |
+ | |||
+ | Assume for the sake of contradiction that there exists a solution over the positive integers. First, the equation implies that <math>2x^2=3(x+y^2)</math>; clearly then <math>3|x</math>, so let <math>x=3a</math>. Then the equation, after simplification, becomes <math>6a^2=3a+y^2</math>. Now <math>3|y</math> from this equation, so let <math>y=3b</math>; therefore, <math>2a^2=a+3b^2</math> for positive integers <math>a</math> and <math>b</math>. | ||
+ | |||
+ | Solving the quadratic in <math>a</math> yields | ||
+ | <cmath>a=\frac{1\pm\sqrt{1+24b^2}}{4}</cmath> | ||
+ | Clearly, for <math>a</math> to be rational, let alone an integer, <math>\sqrt{1+24b^2}</math> must be an integer. Let <math>z=\sqrt{1+24b^2}</math>; then <math>z^2-24b^2=1</math>. This is the Pell Equation for <math>n=24</math>. Testing values yields the minimal solution <math>(z_1,b_1)=(5,1)</math>. (Also notice that <math>z^2=24b^2+1\ge24+1=25</math> since <math>b</math> is a positive integers, hence <math>z\ge5</math>.) Then all solutions <math>z_k</math> in <math>(z_k,b_k)</math> can be generated by the recurrence | ||
+ | <cmath>z_k=z_1z_{k-1}+24b_1b_{k-1}</cmath> | ||
+ | Taking the equation <math>\pmod{4}</math>: | ||
+ | <cmath>z_k\equiv z_1z_{k-1}\pmod{4}</cmath> | ||
+ | We showed earlier that <math>z_1=5</math>; hence, | ||
+ | <cmath>z_k\equiv 5z_{k-1}\equiv z_{k-1}\pmod{4}</cmath> | ||
+ | for all integers <math>k\ge2</math>. Since <math>z_1\equiv1\pmod{4}</math>, we can use the recurrence to conclude that | ||
+ | <cmath>z_k\equiv1\pmod{4}</cmath> | ||
+ | for all such <math>k</math>. Therefore, all integer values of the square root must have this be true. Thus, | ||
+ | <cmath>a=\frac{1\pm z_k}{4}</cmath> | ||
+ | But notice that the numerator must be positive, and since <math>z_k</math> is an integer, the <math>\pm</math> must be a <math>+</math>, so | ||
+ | <cmath>a=\frac{1+z_k}{4}</cmath> | ||
+ | However, <math>1+z_k\equiv2\pmod{4}</math>, so the numerator is not divisible by the denominator; thus <math>a</math> is not a positive integer, which is a contradiction; thus there exist no solutions over the positive integers for <math>a</math> and <math>b</math>, and we are done. | ||
+ | |||
+ | <i>Note: This problem is likely wrong.</i> | ||
+ | |||
+ | ~ [https://artofproblemsolving.com/wiki/index.php/User:Eevee9406 eevee9406] | ||
== See also == | == See also == | ||
https://www.oma.org.ar/enunciados/ibe4.htm | https://www.oma.org.ar/enunciados/ibe4.htm |
Latest revision as of 18:43, 14 April 2025
Problem
Prove that there's and infinity of pairs of natural numbers that satisfy the equation:
~translated into English by Tomas Diaz. ~orders@tomasdiaz.com
Solution
We claim that there do not exist any solutions to this equation over the positive integers.
Assume for the sake of contradiction that there exists a solution over the positive integers. First, the equation implies that ; clearly then
, so let
. Then the equation, after simplification, becomes
. Now
from this equation, so let
; therefore,
for positive integers
and
.
Solving the quadratic in yields
Clearly, for
to be rational, let alone an integer,
must be an integer. Let
; then
. This is the Pell Equation for
. Testing values yields the minimal solution
. (Also notice that
since
is a positive integers, hence
.) Then all solutions
in
can be generated by the recurrence
Taking the equation
:
We showed earlier that
; hence,
for all integers
. Since
, we can use the recurrence to conclude that
for all such
. Therefore, all integer values of the square root must have this be true. Thus,
But notice that the numerator must be positive, and since
is an integer, the
must be a
, so
However,
, so the numerator is not divisible by the denominator; thus
is not a positive integer, which is a contradiction; thus there exist no solutions over the positive integers for
and
, and we are done.
Note: This problem is likely wrong.